Math, asked by kaur401883, 1 year ago

prove that sin theta ( 1+ tan theta) + cos theta (1+ cot theta ) = sec theta +cos theta​

Answers

Answered by Avengers00
25
\underline{\underline{\Huge{\textbf{Question:}}}}

\mathsf{Prove\: that\: sin\, \theta(1+tan\, \theta)+cos\, \theta(1+cot\, \theta) = sec\, \theta+cos\, \theta}


\\

\underline{\underline{\Huge{\textbf{Solution:}}}}

\underline{\LARGE{\textsf{Step-1:}}}
\textsf{Consider LHS}

\underline{\Large{\textbf{LHS = }}}

\implies \mathsf{sin\, \theta(1+tan\, \theta)+cos\, \theta(1+cot\, \theta)}


\\

\underline{\LARGE{\textsf{Step-2:}}}
\textsf{Express tan and cot in terms of}\\\textsf{sin and cos and simplify}

\LARGE{\boxed{\mathbf{\begin{aligned}\quad\bigstar\; \; tan\, \theta &= \dfrac{sin\, \theta}{cos\, \theta}\\\\\quad \bigstar\; \; \cot\, \theta &= \dfrac{cos\, \theta}{sin\, \theta}\quad\end{aligned}}}}

\implies \mathsf{sin\, \theta\left(1+\dfrac{sin\, \theta}{cos\, \theta}\right)+cos\, \theta\left(1+\dfrac{cos\, \theta}{sin\, \theta}\right)}\\\\\implies \mathsf{sin\, \theta\left(\dfrac{cos\, \theta + sin\, \theta}{cos\, \theta}\right)+cos\, \theta\left(\dfrac{sin\, \theta+ cos\, \theta}{sin\, \theta}\right)}\\\\\implies \mathsf{\dfrac{sin\, \theta}{cos\, \theta}(sin\, \theta+cos\, \theta)+\dfrac{cos\, \theta}{sin\, \theta}\left(sin\, \theta+ cos\, \theta\right)}


\\

\underline{\LARGE{\textsf{Step-3:}}}
\textsf{Factor $(sin\, \theta +cos\, \theta)$ from the two terms and simplify}

\implies \mathsf{(sin\, \theta+cos\, \theta)\, \left(\dfrac{sin\, \theta}{cos\, \theta}+ \dfrac{cos\, \theta}{sin\, \theta}\right)}\\\\\implies \mathsf{(sin\, \theta+cos\, \theta)\, \left(\dfrac{sin^2\, \theta+cos^\, \theta}{cos\, \theta\: sin\, \theta}\right)}


\textsf{Using the Identity}
\LARGE{\boxed{\quad\bigstar\; \; \mathbf{sin^2\, \theta+cos^2\, \theta = 1}\quad}}


\implies \mathsf{(sin\, \theta+cos\, \theta)\, \left(\dfrac{1}{cos\, \theta\: sin\, \theta}\right)}\\\\\implies \mathsf{\dfrac{sin\, \theta+cos\, \theta}{cos\, \theta\: sin\, \theta}}\\\\\implies \mathsf{\dfrac{\cancel{sin\, \theta}}{cos\, \theta\: \cancel{sin\, \theta}}+\dfrac{\cancel{cos\, \theta}}{\cancel{cos\, \theta}\: sin\, \theta}}\\\\\implies \mathsf{\dfrac{1}{cos\, \theta}+\dfrac{1}{sin\, \theta}}

\textsf{we have}
\LARGE{\boxed{\mathbf{\begin{aligned}\quad\bigstar\; \; sec\, \theta &= \dfrac{1}{cos\, \theta}\\\\\quad \bigstar\; \; cosec\, \theta &= \dfrac{1}{sin\, \theta}\quad\end{aligned}}}}

\implies \mathsf{sec\, \theta+cosec\, \theta}

\underline{\Large{\textbf{= RHS}}}


\underline{\LARGE{\textbf{Hence Proved}}}

nain31: cool
Avengers00: thank you :)
Similar questions